Jump to content
Urch Forums

bonz103

Members
  • Posts

    26
  • Joined

Everything posted by bonz103

  1. To all the gmat gurus, i have trouble solving DS q's involving number properties in under 2 minutes b/c i try to plug diff numbers in (which sometimes takes me some time)... was wondering if you can provide me with some pointers, guides, etc. to improve my accuracy and speed? any help greatly appreciated! thx guys!
  2. can someone pls also email me the soft copy of the manhattan sc to benji_hsu@hotmail.com ? thx!
  3. pls explain: A survey of entrepreneurs who started companies last year shows that while virtually all did substantial preparatory research and planning, only half used that work to produce a formal business plan. Since, on average, the entrepreneurs w/o formal plans secured the capital they needed in half the time of those with plans, these survey results indicate that, in general, formal plans did not help the entrepreneurs who produced them to secure the capital they needed. Which of the following most seriously weakens the argument? A. Companies started by entrepreneurs who had used formal biz plans to attract investments were on teh whole as profitable in their first year as were companies started by entrepreneurs who had not produced such plans B. In surveys of entrepreneurs who have attempted w/o success to raise sufficient capital, more than half of the respondents indicate that they have produced a formal business plan C. Among the entrepreneurs surveyed, those who did not produce formal business plans sought and received a much larger proportion of their capital from investors with whom they had long-standing biz relationship D. The entrepreneurs surveyed who didn't produce a formal biz plan spent nearly as much time donig preparatory research and planning as the entrepreneurs who produced plans E. The entrepreneurs who produced business plans generally reported later that the process of writing the plan had increased their confidence that their company would succeed. OA
  4. got this right, but want to see how u guys would solve it. thx! Working simultaneously and independently at an identical constant rate, 4 machines of a certain type can produce a total of x units of product P in 6 days. How many of these machines, working simultaneously and independently at this constant rate, can produce a total of 3x units of product P in 4 days? A. 24 B. 18 C. 16 D. 12 E. 8 OA
  5. awesome score! congrats!! best of luck on your apps
  6. one more for D. argument states that number of potential employees will decrease; however, D clearly states the opposite (workforce has increased due to a greater # of workers who are not head of the households). hence, D weakens the conclusion above.
  7. Sorry, can someone pls explain the OA? thx For one toss of a certain coin, the probability that the outcome is heads is 0.6. If the coin is tossed 5 times, which of the following is the probability that the outcome will be heads at least 4 times? A. 0.6^5 B. 2 * (0.6)^4 C. 3 * (0.6)^4 *(0.4) D. 4 * (0.6)^4 * (0.4) + (0.6)^5 E. 5 * (0.6)^4 * (0.4) + (0.6)^5 OA:
  8. can someone pls explain how you would derive the OA? thx the sides of a square region, measured to the nearest cm, are 6 cm long. The least possible value of the actual area of the square region is: A. 36.00 B. 35.00 C. 33.75 D. 30.25 E. 25.00 OA:
  9. pls explain - thx Q1 In Gandania, where government has a monopoly on tobacco sales, the incidence of smoking-related health problems has risen steadily for the last twenty years. The health secretary recently proposed a series of laws aimed at curtailing tobacco use in Gandania. Profits from tobacco sales, however, account for 10% of Gandania's annual revenues. Therefore, Gandania cannot afford to institute the proposed laws. Which of the following, if true, most seriously weakens the argument? A. All health care in Gandania is gov.-funded B. Implementing the proposed laws is not likely to cause a sig increase in the amt of tobacco Gandania exports C. The % of rev Gandania receives from tobacco sales has remained steady in recent years D. Profits from tobacco sales far surpass any other single source of revenues for the Gandanian gov E. No gov official in Gandania has ever previously proposed laws aimed at curtailing tobacco use Q2. Which of the following most logically completes the editorial below? Editorial in G Newspaper: For almost 3 months, opposition parties have been mounting daily street demonstrations in the capital in an effort to pressure the ruling party into calling an election. Though the demonstrations were well attended at first, attendance has declined steadily in recent weeks. However, the decline in attendance does not indicate that popular support for the opposition's demands is dropping, since ____ A. the opposition's demands have not changed during the period when the street demonstrations have been mounted B. no foreign govs have expressed any support for the opposition's demands C. The state-controlled media have ceased any mention of the demonstrations, leaving many ctiizens outside the capital with no way of knowing that demonstrations continue D. There have not recently been any anti-gov demonstrations in cities other than the capital E. a recent sharp decrease in unemployment has led to increased popular support for the gov Q3: Excavations of the Roman city of Sepphoris have uncovered numerous detailed mosaics depicting several readily identifiable animal species: a hare, a patridge, and various Mediterranean fish. Oddly, most of the species represented did not live in the Sepphoris region when these mosaics were created. Since identical motifs appear in mosaics found in other Roman cities, however, the mosaics of Sepphoris were very likely created by traveling artisans from some other part of the Roman empire. Which of the following is an assumption on which the argument depends? A. The Sepphoris mosaics are not composed exclusively of types of stones found naturally in teh Sepphoris area B. There is no single region to which all the species depeicted in the Sepphoris mosaics are native C. No motifs appear in the Sepphoris mosaics that do not also appear in the mosaics of some other Roman city D. All of the animal figures in the Sepphoris mosiacs are readily identifiable as representation of known species E. There was not a common repertory of mosaic designs with which artisans who lived in various parts of the Roman empire were familiar OA:
  10. sorry, this may have been brought up before, but i couldn't find the thread so am posting again. still don't know how to reach the ans. thks the function f is defined for each positive three-digit integer n by f(n) = 2^x * 3^y * 5^z, where x, y, and z are the hundreds, tens and units digits of n, respectively. If m and v are three-digit positive integers such that f(m) = 9 * f(v), then m-v = A. 8 B. 9 C. 18 D. 20 E. 80 OA:
  11. don't have the OA, but wld appreciate your thoughts on the ans: Q: Lyme disease is caused by a bacterium transmitted to humans by deer ticks. Generally deer ticks pick up the bacterium while in the larval stage from feeding on infected white-footed mice. However, certain other species on which the larvae feed do not harbor the bacterium. Therefore, if the population of these other species were increased, the number of ticks acquiring the bacterium and hence the number of people contracting Lyme disease would likely decline. Which of the following, if true, most strengthens the argument? A) Ticks do not suffer any adverse consequences from carrying the bacterium that causes Lyme disease in humans. B) There are no known cases of a human’s contracting Lyme disease through contact with white-footed mice. C) A deer tick feeds only once while in the larval stage. D) A single host animal can be the source of bacterium for many tick larvae. E) None of the other species on which deer tick larvae feed harbor other bacteria that ticks transmit to humans
  12. IMO C - the fact that 3 out of 4 hospitals administer patients Novex can be explained by an alternative reason (i.e. drug manufactuers sell to hospitals the cheapest drugs to earn profit), which weakens the original argument that Novex is the most effective painkiller for sinus pain since 3 out of 4 hospitals give their patients this drug
  13. IMO C the argument is saying that the pharma's division is growing stronger; this is supported by evidence of an increase in profits. however, an increase in profits may not necessarily mean the pharma division is growing stronger.
  14. Actual - 640 Practice Tests: GMATPrep 1: 690 GMATPrep 2: 710 Princeton Review CAT 1: 680 CAT 2: 670 CAT 3: 690 i guess i'm the only person that didn't get a score in-line with my gmatprep results...
  15. just took the gmat a couple of min ago and was shocked to see my low score. i was very confident i would do well (given my recent mock exam scores), so was very depressed upon learning i was far from my target range :( . given i am currently in-between jobs in the finance industry, this was my only shot to do well given i had a 1.5 months to prepare and once i start working i will not have the time to study. In addition, this is my second time taking the exam, and the improvement has been very minimal. given apps are due in the next 2 months, i don't know whether i shld continue with the app process or just concentrate on my new job (given i only want to apply to a handful of schools). any advice much appreciated. most imptly, want to thank everyone on this forum as you've all helped me a great deal in the past month! hope i can also post something positive next time... below is a summary of my mock exams: Real GMAT 1st - 600 (44 Q, 28 V) 2nd - 640 (43 Q, 35 V) GMATPrep 1 - 690 (48 Q, 37 V) --> last week GMATPrep 2 - 710 (50 Q, 39 V) --> taken 2 days ago Princeton Review CAT CAT 1 - 680 (48 Q, 37 V) CAT 2 - 670 (47 Q, 35 V) CAT 3 - 690 (50 Q, 38 V)
  16. can someone pls explain the logic as to why the OA is the right ans? thks When people predict that certain result will not take place unless a certain action is taken, they believe that they have learned that the prediction is correct when the action is taken and the result occurs. On reflection, however, it often becomes clear that the result admits of more than one interpretation. Which of the following, if true, best supports the claims above? (A) Judging the success of an action requires specifying the goal of the action. (B) Judging which action to take after a prediction is made requires knowing about other actions that have been successful in similar past situations. © Learning whether a certain predictive strategy is good requires knowing the result using that strategy through several trials. (D) Distinguishing a correct prediction and effective action from an incorrect prediction and ineffective action is often impossible. (E) Making a successful prediction requires knowing the facts about the context of that prediction.
  17. sorry, don't have any OAs, but ur thoughts greatly appreciated. thx! Q1 If x=3y, is x^2>y^2? (1) y+x (2) X^2=9y^2 Q2 In the xy-plane, line L passes through point (1,p). Does L have a positive slope? 1). L passes through point (-p, 13) 2). L passes through point (0,1) Q3 For integer n, f(n) denotes the remainder when n is divided by integer k. Is k greater than 10? 1). f(k+32)=8 2). f(k+42)=6 Q4 If x and y are integers, |x|*|y|=? 1). xy=10 2). x*|y|=-10 Q5 Symbol © denotes that when integer K is even, ©K=K-1, when integer K is odd, ©K=K+1. If a and b are integers, is ©a*©b even? 1). a is odd 2). b is odd
  18. sorry, i received these q's w/o any OAs or answer choices; nevertheless, i think these are good q's that i want to share w/ everyone. pls kindly discuss. thks! 1) 5 people are to be seated around a circular table. Two seating arrangements are considered different only when the positions of the people are different relative to each other. What is the total number of different possible seating arrangements for the group? 2) A three-digit code consists of digits from 0 to 9. If the first digit cannot be 0 or 1, the second digit must be 0 or 1, and the second and third digits cannot be 0 at the same time, how many codes are possible? 3) DS: What is the remainder when positive integer x is divided by 7 1). X+1 is divisible by 7 2). X+13 is divisible by 7 4) A company has assigned a distinct 3-digit code number to each of its 330 employees. Each code number was formed from the digits 2, 3, 4, 5, 6, 7, 8, 9 and no digit appears more than once in any one code number. How many unassigned code numbers are there? 5) In which quadrants do the roots of the equation lie, given x and y are integers: Y = (x-2)^2 -1 6) K=wxyz, where w, x, y, z are prime numbers. Not including 1 and K, how many factors does K have? 7) DS:Symbol [d] denotes the least integer greater than or equal to d, is [2d]=0? (1) [d]= 0 (2) [3d] = 0 8) How many arrangements of A, B, C, D, and E are there, such that B and D are not adjacent? Each letter is used exactly once 9) DS: For integer n, f(n) denotes the remainder when n is divided by integer k. Is k greater than 10? 1). f(k+32)=8 2). f(k+42)=6
  19. 1) m = 35 2) i got hypotenuse = 16 or B
  20. was wondering if someone can give me a hand on how to approach this... this is from the gmatprep test. thks! Q: A boat traveled upstream a distance of 90 miles at an average speed of v-3 mph and then traveled the same distance downstream at an average speed of v+3 mph. If the upstream took half an hr longer than the trip downstream, how many hours did the boat travel downstream? A. 2.5 B. 2.4 C. 2.3 D. 2.2 E. 2.1
  21. 1. Fact: Asthma, a bronchial condition, is much less common ailment than hay fever, an allergic inflammation of the nasal passages. Fact: Over 95 percent of people who have asthma also suffer from hay fever. If the information given as facts above is true, which of the following must also be true? A. Hay fever is a prerequisite for the development of asthma. B. Asthma is a prerequisite for the development of hay fever. C. Those who have neither hay fever nor asthma comprise less than 5 percent of the total population. D. The number of people who have both of these ailments is greater than the number of people who have only one of them. E. The percentage of people suffering from hay fever who also have asthma is lower than 95 percent. 2. Unless tiger hunting decreases, tigers will soon be extinct in the wild. The countries in which the tigers’ habitats are located are currently debating joint legislation that would ban tiger hunting. Thus, if these countries can successfully enforce this legislation, the survival of tigers in the wild will be ensured. The reasoning in the argument is most vulnerable to criticism on the grounds that the argument assumes without sufficient warrant that a ban on tiger hunting could be successfully enforced considers the effects of hunting on tigers without also considering the effects of hunting on other endangered animal species fails to take into account how often tiger hunters are unsuccessful in their attempts to kill tigers neglects to consider the results of governmental attempts in the past to limit tiger hunting takes the removal of an impediment to the tigers’ survival as a guarantee of their survivalQ3 Telomerase is an enzyme that is produced only in cells that are actively dividing. For this reason it is generally absent from body tissues in adults. Bone marrow is an exception to this rule, however, since even in adults, bone marrow cells continually divide to replace old blood cells. Cancers are another exception, because their cells are rapidly dividing. The information provided most strongly supports which of the following? Telomerase is the only enzyme that is present in cancerous cells but absent from cells that are not actively dividing. In children, the only body tissues from which telomerase is absent are those in which cells are not rapidly dividing. The presence of telomerase in bone marrow is no indication of bone marrow cancer. Cancer of the bone marrow develops more rapidly than cancer growing in any other kind of adult tissue. The level of telomerase production is always higher in cancerous tissue than in noncancerous tissue.
  22. another way you can think of it is given you know QS = 5 due to the 3, 4, 5 right triangle property, treat triangles RQP and RSP separately and solve for the common side (PR) using the pyth theorm i.e. if QR = x, RS = 5-x and PR = y x^2 + y^2 = 9 (5-x)^2 + y^2 = 16 solve for y
  23. again, any explanations would be much appreciated - thks! Q1 A photographer will arrange 6 people of 6 different heights for photograph by placing them in two rows of three so that each person in the first row is standing in front of someone in the second row. The heights of the people within each row must increase from left to right, and each person in the second row must be taller than the person standing in front of him or her. How many such arrangements of the 6 people are possible? A. 5 B. 6 C. 9 D. 24 E. 36 Q2: Running at their respective constant rates, machine X takes 2 days longer to produce w widgets than machine Y. At these rates, if the two machines together produce 5/4 w widgets in 3 days, how many days would it take machine X alone to produce 2w widgets? A. 4 B. 6 C. 8 D. 10 E. 12 Q3: June 25, 1982, fell on a Friday. On which day of the week did June 25, 1987, fall? (Note: 1984 was a leap year.) A. Sunday B. Monday C. Tuesday D. Wednesday E. Thursday
  24. thks so much - v helpful!
  25. thks for your help! these q's are part of a problem set that was compiled by another online user... was told some of these q's appeared on actual exams in the past...
×
×
  • Create New...